Profile Picture

Multi series line chart

Posted By Jakub Rehacek 3 Years Ago
Author
Message
Jakub Rehacek
Question Posted 3 Years Ago
View Quick Profile
Forum Newbie

Forum Newbie (0 reputation)Forum Newbie (0 reputation)Forum Newbie (0 reputation)Forum Newbie (0 reputation)Forum Newbie (0 reputation)Forum Newbie (0 reputation)Forum Newbie (0 reputation)Forum Newbie (0 reputation)Forum Newbie (0 reputation)

Group: Forum Members
Last Active: 2 Years Ago
Posts: 10, Visits: 69
I need to generate a date based line chart with multiple series.
The number of series will vary between 2 to 30. Is there a way to do this in the code, since the number of series on the char is dependent on the outcome of a query. So

Dim line1 As NLineSeries = CreateLineSeries(Color.Green, Color.DarkOliveGreen)
Dim line2 As NLineSeries = CreateLineSeries(Color.Red, Color.DarkRed)


will not work

Can I use

For i=1 to MaxSeries
    Dim line(i) As NLineSeries = CreateLineSeries(Color.Green, Color.DarkOliveGreen)
Next I


Or something like that?

Thanks





Nevron Support
Posted 3 Years Ago
View Quick Profile
Supreme Being

Supreme Being (4,329 reputation)Supreme Being (4,329 reputation)Supreme Being (4,329 reputation)Supreme Being (4,329 reputation)Supreme Being (4,329 reputation)Supreme Being (4,329 reputation)Supreme Being (4,329 reputation)Supreme Being (4,329 reputation)Supreme Being (4,329 reputation)

Group: Forum Members
Last Active: Last Year
Posts: 3,039, Visits: 3,746
Hi Jakub,

There is no problem to add as many series to the chart as required - for example:

Dim chart As NChart = nChartControl1.Charts(0)

For i = 0 To 10
Dim line As New NLineSeries()
chart.Series.Add(line)

' do something with line series

Next

will add ten new series to the chart. We hope this helps - let us know if you have any questions or meet any problems.


Best Regards,
Nevron Support Team



Jakub Rehacek
Posted 3 Years Ago
View Quick Profile
Forum Newbie

Forum Newbie (0 reputation)Forum Newbie (0 reputation)Forum Newbie (0 reputation)Forum Newbie (0 reputation)Forum Newbie (0 reputation)Forum Newbie (0 reputation)Forum Newbie (0 reputation)Forum Newbie (0 reputation)Forum Newbie (0 reputation)

Group: Forum Members
Last Active: 2 Years Ago
Posts: 10, Visits: 69
Thanks.
I got it working.
But, why do I get lines connecting end of one series to the beginning of the next one? Number 19 is last point in the first series, and number 3 is the first point in the second series, but they are connected by a line. ??
Also, how can get markers to display in the legend?

Thx.
data:image/png;base64,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


Nevron Support
Posted 3 Years Ago
View Quick Profile
Supreme Being

Supreme Being (4,329 reputation)Supreme Being (4,329 reputation)Supreme Being (4,329 reputation)Supreme Being (4,329 reputation)Supreme Being (4,329 reputation)Supreme Being (4,329 reputation)Supreme Being (4,329 reputation)Supreme Being (4,329 reputation)Supreme Being (4,329 reputation)

Group: Forum Members
Last Active: Last Year
Posts: 3,039, Visits: 3,746
Hi Jakub,

Can you post the code you're using to create this chart for review? To show markers in the legend the line series must have markers enabled as well - for example:

someLineSeries.MarkerStyle.Visible = True
someLineSeries.MarkerStyle.FillStyle = New NColorFillStyle(Color.Red)
someLineSeries.MarkerStyle.Width = New NLength(4)
someLineSeries.MarkerStyle.Height = New NLength(4)

Let us know if you have any questions.

Best Regards,
Nevron Support Team



Jakub Rehacek
Posted 3 Years Ago
View Quick Profile
Forum Newbie

Forum Newbie (0 reputation)Forum Newbie (0 reputation)Forum Newbie (0 reputation)Forum Newbie (0 reputation)Forum Newbie (0 reputation)Forum Newbie (0 reputation)Forum Newbie (0 reputation)Forum Newbie (0 reputation)Forum Newbie (0 reputation)

Group: Forum Members
Last Active: 2 Years Ago
Posts: 10, Visits: 69
Thanks, I have figured the issue with continuous lines. It was my mistake in defining the series.
I know how to enable the markers, but they are all the same marker.
I know that I can assign a  PointShape to the marker, but how do I do it for a random number of series (2-30)?
I do not care what the markers are, just that they are different shape and color for each series.
Is there a numeric representation of PointShapes that I could use while cycling through the recordset?
Same for the FillStyle?

Thx.
data:image/png;base64,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


Jakub Rehacek
Posted 3 Years Ago
View Quick Profile
Forum Newbie

Forum Newbie (0 reputation)Forum Newbie (0 reputation)Forum Newbie (0 reputation)Forum Newbie (0 reputation)Forum Newbie (0 reputation)Forum Newbie (0 reputation)Forum Newbie (0 reputation)Forum Newbie (0 reputation)Forum Newbie (0 reputation)

Group: Forum Members
Last Active: 2 Years Ago
Posts: 10, Visits: 69

I was mistaken. The lines are still connecting between the series. See below There should be no connecting line between 109 and 1 as those are two different series. See the underlying data in the grid below.
I've attached a snipped of code. I am looping through the series in the first sub, adding the series to the chat, from within that loop I am calling another sub to iterate through values (and dates) within the series and adding them to the chart.
I figured how to add different markers to the legend. But they are not modified in the chart. Event though I am passing the same integer to the Markerstyle.PointShape in the subroutine
data:image/png;base64,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





Sub1
        sda.Fill(dt)

        Dim chart As NChart = NChartControl1.Charts(0)
        chart.LabelLayout.EnableInitialPositioning = True
        Dim rnd As New Random

        Dim OldSeries As String = ""
        Dim NewSeries As String = ""


        For i As Integer = 0 To dt.Rows.Count - 1

            NewSeries = dt.Rows(i)(0).ToString


            Dim line As New NLineSeries()
   


            chart.Series.Add(line)

                line.MultiLineMode = MultiLineMode.Series
                line.Legend.TextStyle.FontStyle.EmSize = New NLength(8)
                line.DataLabelStyle.Visible = True
                line.Name = NewSeries
                line.LineSegmentShape = LineSegmentShape.Line


                line.UseXValues = True
             
                line.BorderStyle.Color = Color.Black
                line.DataLabelStyle.Visible = True
                line.InflateMargins = True
                line.MarkerStyle.Visible = True
                line.MarkerStyle.BorderStyle.Color = Color.Black
               
                line.MarkerStyle.FillStyle = New NColorFillStyle(Color.FromArgb(255, rnd.Next(255), rnd.Next(255), rnd.Next(255)))
                line.MarkerStyle.PointShape = i
                line.MarkerStyle.Width = New NLength(2, NRelativeUnit.ParentPercentage)
                line.MarkerStyle.Height = New NLength(2, NRelativeUnit.ParentPercentage)
      

               
                UpdateTimeSpan(NewSeries, i)


        Next

        ' apply style sheet
        Dim styleSheet As NStyleSheet = NStyleSheet.CreatePredefinedStyleSheet(PredefinedStyleSheet.FreshMultiColor)
        styleSheet.Apply(NChartControl1.Document)


        UpdateDateTimeScale()
       

    End Sub
    Private Sub UpdateTimeSpan(Test As String, Pnt As Integer)
        Dim myConnection As New SqlConnection(strConnString)
        Dim myReader As SqlDataReader = Nothing
        Dim dt As New DataTable()
        Dim con As New SqlConnection(strConnString)
        Dim sda As New SqlDataAdapter()
        Dim cmd As New SqlCommand("GetProductionTestDaily")
        cmd.CommandType = CommandType.StoredProcedure

        Dim dteFrom As Date
        Dim dteTo As Date
        Dim txtTest As String

        Dim startDate As DateTime = DateTime.Parse(Me.txtdteFrom.Text)
        dteTo = DateAdd(DateInterval.Day, 1, CDate(Me.txtdteTo.Text)).ToString

        Dim endDate As DateTime

        Dim rnd As New Random

        endDate = dteTo

        dteFrom = Me.txtdteFrom.Text
        txtTest = Me.ddTest.SelectedValue


        cmd.Parameters.AddWithValue("@dteFrom", dteFrom)
        cmd.Parameters.AddWithValue("@dteTo", dteTo)
        cmd.Parameters.AddWithValue("@Test", Test)



        cmd.Connection = con
        sda.SelectCommand = cmd



        If startDate > endDate Then
            Dim temp As DateTime = startDate

            startDate = endDate
            endDate = temp
        End If

        ' Get the line series from the chart
        Dim chart As NChart = NChartControl1.Charts(0)
        Dim line As NLineSeries = CType(chart.Series(0), NLineSeries)



        Dim span As TimeSpan = endDate.Subtract(startDate)
        span = New TimeSpan(span.Ticks / 30)
        Dim span1 As TimeSpan '= span.Days


        line.UseXValues = True



        sda.Fill(dt)


        If span.Ticks > 0 Then
           For i As Integer = 0 To dt.Rows.Count - 1

                dteTo = dt.Rows(i)(0)
                span1 = dteTo.Subtract(startDate)


                startDate = startDate.AddDays(span1.Days)


                line.XValues.Add(startDate)
                line.Values.Add(dt.Rows(i)(1))
                line.MarkerStyle.PointShape = Pnt


            Next

      
        End If



    End Sub





Nevron Support
Posted 3 Years Ago
View Quick Profile
Supreme Being

Supreme Being (4,329 reputation)Supreme Being (4,329 reputation)Supreme Being (4,329 reputation)Supreme Being (4,329 reputation)Supreme Being (4,329 reputation)Supreme Being (4,329 reputation)Supreme Being (4,329 reputation)Supreme Being (4,329 reputation)Supreme Being (4,329 reputation)

Group: Forum Members
Last Active: Last Year
Posts: 3,039, Visits: 3,746
Hi Jacub,

Please note that you also modify the marker style poin shape with:

line.MarkerStyle.PointShape = Pnt

at the end of the source code, which is probably a constant defined somewhere else. 

Best Regards,
Nevron Support Team





Similar Topics


Reading This Topic